Докажите что при любых a,b,c уравнение имеет 2 решения или не имеет их...

0 голосов
50 просмотров

Докажите что при любых a,b,c уравнение имеет 2 решения или не имеет их вообще.
x^4+(2a+1-b^2)*x^3+a*x^2+|c-|a|^b |*x+|c+|b|^a |+1=0


Алгебра (11.7k баллов) | 50 просмотров
0

Перезагрузи страницу если не видно

0

Погодите sqrt(2a+1-b^2)!!!! Я забыл!

0

Так вы не решите. Это очень важно!!!!

0

я не забыл

0

Ого да наши пути различаются!!!!

0

Хорошее решение. Хотите узнать решение короче

0

Давайте

0

Лучше пишите в ЛС

0

На самом деле (x1+x2+x3+x4)^2=x1^2+x2^2+x3^2+x4^2+2x1x2+2x1x3+2x1x4+2x2x3+2x2x4+2x3x4 2a+1-b^2=(x1^2+x2^2+x3^2+x4^2)+2a

0

x1^2+x2^2+x3^2+x4^2=1-b^2<=1 откуда тк квадраты положительны то квадрат каждого корня по модулю меньше единици,то все корни меньше единици. А тогда их произведение по модулю меньше единици. НО произведение корней больше 1. Тк последний член всегда больше 1. То есть противоречие!!!

Дан 1 ответ
0 голосов
Правильный ответ

    Положим что  корни уравнения равны   x_{1};x_{2};x_{3} ; x_{4} 
 Тогда их сумма  равна    -\sqrt{2a+1-b^2} это 
  x^4+\sqrt{2a+1-b^2}x^3+ax^2-(c+|b|^a)x+|c-|a|^b|+1=0 \\\\
x_{1}+x_{2}+x_{3}+x_{4}=-\sqrt{2a+1-b^2}\\
x_{1}x_{2}+x_{1}x_{3}+x_{1}x_{4}+x_{2}x_{3}+x_{2}x_{4}+x_{3}x_{4}=a\\
x_{1}x_{2}x_{3}+x_{1}x_{2}x_{4} + x_{1}x_{3}x_{4}+x_{2}x_{3}x_{4}=|c+|a|^b|\\
x_{1}x_{2}x_{3}x_{4}=|c-|b|^a|+1\\\\ 
 
 
 Заметим что  сумма корней отрицательное число ,   а произведение корней  всегда положительное    число , значит   
 Либо два корня отрицательны , либо все  корни отрицательны 
 x_{1},x_{2} , x_{3},x_{4} \neq 0\\\\
 
Рассмотрим    второй случаи 
Если   x_{1},x_{2}<0\\
 без потери общности   можно взять imagex_{4}>0" alt="x_{3}>x_{4}>0" align="absmiddle" class="latex-formula"> 
 Из первого image-\frac{1}{2}" alt=" b \in [-\sqrt{2a+1};\sqrt{2a+1} ] \\ a>-\frac{1}{2}" align="absmiddle" class="latex-formula"> 
Из третьего  так как произведение всех корней отрицательно , значит  сумма   S<0  , но это не верно , так как стоит модуль , значит четыре корня   не может быть. 
Второй случаи ,  возможен , но не всегда  
 x_{1};x_{2}<0\\
  по второму условию следует что 
  image0" alt="a>0" align="absmiddle" class="latex-formula"> 
 По третьему 
 image0" alt=" x_{1}x_{2}x_{3}>0" align="absmiddle" class="latex-formula"> 
  Возможно когда  imagex_{1}x_{2}x_{4}+x_{3}x_{4}+x_{2}x_{3}x_{4}" alt="x_{1}x_{2}x_{3} >x_{1}x_{2}x_{4}+x_{3}x_{4}+x_{2}x_{3}x_{4}" align="absmiddle" class="latex-formula"> 

  

  
 

(224k баллов)